Example involving the Chinese Remainder Theorem











up vote
1
down vote

favorite
1












I am working on a Number Theory book and I have come across the following problem:



(Underwood Dudley 2nd Edition Section 5 Problem 3):



Solve the system:



x $equiv 3(mod 5)$



x $equiv 5(mod7)$



x $equiv 7(mod11)$



I understand that I must use the Chinese Remainder theorem and I understand that the CRT states that if the GCD of these three numbers there exists a unique solution (mod 385), but my book gives me no instruction on how to go about finding this solution--other than cold hard calculation. Could I have some advice or direction on the method to solving this problem and the idea behind the method? Thank you!










share|cite|improve this question


















  • 1




    Hmm..., it appears that 'the' technique to solve such systems is not illustrated in the book and instead is given as exercise 14. So I think the best you can do is to adapt the solution which is given to an example. (I have an older version of the book in which your problem is problem 4c, page 40.)
    – barto
    Oct 24 '14 at 20:51















up vote
1
down vote

favorite
1












I am working on a Number Theory book and I have come across the following problem:



(Underwood Dudley 2nd Edition Section 5 Problem 3):



Solve the system:



x $equiv 3(mod 5)$



x $equiv 5(mod7)$



x $equiv 7(mod11)$



I understand that I must use the Chinese Remainder theorem and I understand that the CRT states that if the GCD of these three numbers there exists a unique solution (mod 385), but my book gives me no instruction on how to go about finding this solution--other than cold hard calculation. Could I have some advice or direction on the method to solving this problem and the idea behind the method? Thank you!










share|cite|improve this question


















  • 1




    Hmm..., it appears that 'the' technique to solve such systems is not illustrated in the book and instead is given as exercise 14. So I think the best you can do is to adapt the solution which is given to an example. (I have an older version of the book in which your problem is problem 4c, page 40.)
    – barto
    Oct 24 '14 at 20:51













up vote
1
down vote

favorite
1









up vote
1
down vote

favorite
1






1





I am working on a Number Theory book and I have come across the following problem:



(Underwood Dudley 2nd Edition Section 5 Problem 3):



Solve the system:



x $equiv 3(mod 5)$



x $equiv 5(mod7)$



x $equiv 7(mod11)$



I understand that I must use the Chinese Remainder theorem and I understand that the CRT states that if the GCD of these three numbers there exists a unique solution (mod 385), but my book gives me no instruction on how to go about finding this solution--other than cold hard calculation. Could I have some advice or direction on the method to solving this problem and the idea behind the method? Thank you!










share|cite|improve this question













I am working on a Number Theory book and I have come across the following problem:



(Underwood Dudley 2nd Edition Section 5 Problem 3):



Solve the system:



x $equiv 3(mod 5)$



x $equiv 5(mod7)$



x $equiv 7(mod11)$



I understand that I must use the Chinese Remainder theorem and I understand that the CRT states that if the GCD of these three numbers there exists a unique solution (mod 385), but my book gives me no instruction on how to go about finding this solution--other than cold hard calculation. Could I have some advice or direction on the method to solving this problem and the idea behind the method? Thank you!







elementary-number-theory chinese-remainder-theorem






share|cite|improve this question













share|cite|improve this question











share|cite|improve this question




share|cite|improve this question










asked Oct 24 '14 at 20:32









candido

420212




420212








  • 1




    Hmm..., it appears that 'the' technique to solve such systems is not illustrated in the book and instead is given as exercise 14. So I think the best you can do is to adapt the solution which is given to an example. (I have an older version of the book in which your problem is problem 4c, page 40.)
    – barto
    Oct 24 '14 at 20:51














  • 1




    Hmm..., it appears that 'the' technique to solve such systems is not illustrated in the book and instead is given as exercise 14. So I think the best you can do is to adapt the solution which is given to an example. (I have an older version of the book in which your problem is problem 4c, page 40.)
    – barto
    Oct 24 '14 at 20:51








1




1




Hmm..., it appears that 'the' technique to solve such systems is not illustrated in the book and instead is given as exercise 14. So I think the best you can do is to adapt the solution which is given to an example. (I have an older version of the book in which your problem is problem 4c, page 40.)
– barto
Oct 24 '14 at 20:51




Hmm..., it appears that 'the' technique to solve such systems is not illustrated in the book and instead is given as exercise 14. So I think the best you can do is to adapt the solution which is given to an example. (I have an older version of the book in which your problem is problem 4c, page 40.)
– barto
Oct 24 '14 at 20:51










4 Answers
4






active

oldest

votes

















up vote
1
down vote



accepted










Here's a method, sometimes called 'adding the modulus', that works fairly well for small moduli--I'll apply it to your problem:



Start with the congruence of the largest modulus, and as we go through each step, we watch for a number that satisfies any of the remaining congruences.:



$pmod{11}: xequiv 7equiv 18$. We notice that $18$ also satisfies $xequiv 3pmod{5}$.



So $xequiv 18 pmod{55}$.



Then $pmod{55}: xequiv 18equiv 73equiv 128equiv 183equiv 238equiv 293equiv348 $. Here we notice that $348$ also satisfies $xequiv 5pmod{7}$.



Thus our solution is $xequiv 348pmod{385}$






share|cite|improve this answer





















  • @candido: You're welcome. I'm hope this was helpful.
    – paw88789
    Oct 28 '14 at 21:56


















up vote
0
down vote













Since $xequiv -2$ both $pmod5$ and $pmod7$, it must be congruent to $-2equiv 33 pmod{35}$.



Since $x+2equiv 0pmod{35}$, and $x+2equiv 9pmod{11}$, we want to know what $n$ makes $35nequiv 2nequiv 9pmod{11}$. We see that $n=10$ does the trick.



So the answer is $xequiv 35cdot10 -2 = 348pmod{385}$.






share|cite|improve this answer




























    up vote
    0
    down vote













    Let $(a_1, a_2, a_3) = (3, 5, 7)$ and let $(n_1, n_2, n_3) = (5, 7, 11)$ and let:
    $$
    x = c_1a_1 + c_2a_2 + c_3a_3
    $$
    We want to solve for the coefficients $c_i$ so that they have the following special property:
    $$
    c_i equiv begin{cases}
    1 pmod {n_j} &text{if } j = i \
    0 pmod {n_j} &text{if } j neq i \
    end{cases}
    $$
    If we can do this, then $x$ will automatically satisfy the system of congruences! For example, $x equiv 5 pmod 7$ because:
    begin{align*}
    x
    &= 3c_1 + 5c_2 + 7c_3 \
    &equiv 3(0) + 5(1) + 7(0) pmod 7 \
    &equiv 5 pmod 7
    end{align*}
    So it remains to find these magical coefficients. I'll show you how to get $c_2$.





    We want $c_2 equiv 1 pmod 7$ and $c_2 equiv 0 pmod 5$ and $c_2 equiv 0 pmod {11}$. From the last two conditions, we know that $55$ divides $c_2$. Now what multiple of $55$ would have a remainder of $1$ when divided by $7$? By inspection, notice that $-55 = (-8)7 + 1$ [if this step didn't jump out to you, you could use the Extended Euclidean Algorithm to find integers $u,v$ such that $55u + 7v = gcd(55, 7) = 1$, then just take $c_2 = 55u$]. So we can take $c_2 = -55$.





    Try to find the other two coefficients yourself!






    share|cite|improve this answer




























      up vote
      0
      down vote













      This is not a general solution, just a solution based on your specific example.



      $x equiv 3 pmod 5$



      $ x equiv 5 pmod 7$



      $ x equiv 7 pmod{11}$



      First notice that



      $x+2 equiv 0 pmod 5$ and $ x+2 equiv 0 pmod 7$



      Hence $x + 2 equiv 0 pmod{35}$. That is to say, $x = 35n - 2$ for some integer, $n$.



      So begin{align}
      35n - 2 &equiv 7 pmod{11} \
      2n &equiv 9 pmod{11} \
      2n &equiv -2 pmod{11} \
      n &equiv -1 pmod{11}
      end{align}



      So $n = 11m - 1$ and $x = 35(11m -1) - 2 = 385m - 37$



      S0 $x equiv -37 equiv 348 pmod{385}$






      share|cite|improve this answer





















        Your Answer





        StackExchange.ifUsing("editor", function () {
        return StackExchange.using("mathjaxEditing", function () {
        StackExchange.MarkdownEditor.creationCallbacks.add(function (editor, postfix) {
        StackExchange.mathjaxEditing.prepareWmdForMathJax(editor, postfix, [["$", "$"], ["\\(","\\)"]]);
        });
        });
        }, "mathjax-editing");

        StackExchange.ready(function() {
        var channelOptions = {
        tags: "".split(" "),
        id: "69"
        };
        initTagRenderer("".split(" "), "".split(" "), channelOptions);

        StackExchange.using("externalEditor", function() {
        // Have to fire editor after snippets, if snippets enabled
        if (StackExchange.settings.snippets.snippetsEnabled) {
        StackExchange.using("snippets", function() {
        createEditor();
        });
        }
        else {
        createEditor();
        }
        });

        function createEditor() {
        StackExchange.prepareEditor({
        heartbeatType: 'answer',
        convertImagesToLinks: true,
        noModals: true,
        showLowRepImageUploadWarning: true,
        reputationToPostImages: 10,
        bindNavPrevention: true,
        postfix: "",
        imageUploader: {
        brandingHtml: "Powered by u003ca class="icon-imgur-white" href="https://imgur.com/"u003eu003c/au003e",
        contentPolicyHtml: "User contributions licensed under u003ca href="https://creativecommons.org/licenses/by-sa/3.0/"u003ecc by-sa 3.0 with attribution requiredu003c/au003e u003ca href="https://stackoverflow.com/legal/content-policy"u003e(content policy)u003c/au003e",
        allowUrls: true
        },
        noCode: true, onDemand: true,
        discardSelector: ".discard-answer"
        ,immediatelyShowMarkdownHelp:true
        });


        }
        });














        draft saved

        draft discarded


















        StackExchange.ready(
        function () {
        StackExchange.openid.initPostLogin('.new-post-login', 'https%3a%2f%2fmath.stackexchange.com%2fquestions%2f989564%2fexample-involving-the-chinese-remainder-theorem%23new-answer', 'question_page');
        }
        );

        Post as a guest















        Required, but never shown

























        4 Answers
        4






        active

        oldest

        votes








        4 Answers
        4






        active

        oldest

        votes









        active

        oldest

        votes






        active

        oldest

        votes








        up vote
        1
        down vote



        accepted










        Here's a method, sometimes called 'adding the modulus', that works fairly well for small moduli--I'll apply it to your problem:



        Start with the congruence of the largest modulus, and as we go through each step, we watch for a number that satisfies any of the remaining congruences.:



        $pmod{11}: xequiv 7equiv 18$. We notice that $18$ also satisfies $xequiv 3pmod{5}$.



        So $xequiv 18 pmod{55}$.



        Then $pmod{55}: xequiv 18equiv 73equiv 128equiv 183equiv 238equiv 293equiv348 $. Here we notice that $348$ also satisfies $xequiv 5pmod{7}$.



        Thus our solution is $xequiv 348pmod{385}$






        share|cite|improve this answer





















        • @candido: You're welcome. I'm hope this was helpful.
          – paw88789
          Oct 28 '14 at 21:56















        up vote
        1
        down vote



        accepted










        Here's a method, sometimes called 'adding the modulus', that works fairly well for small moduli--I'll apply it to your problem:



        Start with the congruence of the largest modulus, and as we go through each step, we watch for a number that satisfies any of the remaining congruences.:



        $pmod{11}: xequiv 7equiv 18$. We notice that $18$ also satisfies $xequiv 3pmod{5}$.



        So $xequiv 18 pmod{55}$.



        Then $pmod{55}: xequiv 18equiv 73equiv 128equiv 183equiv 238equiv 293equiv348 $. Here we notice that $348$ also satisfies $xequiv 5pmod{7}$.



        Thus our solution is $xequiv 348pmod{385}$






        share|cite|improve this answer





















        • @candido: You're welcome. I'm hope this was helpful.
          – paw88789
          Oct 28 '14 at 21:56













        up vote
        1
        down vote



        accepted







        up vote
        1
        down vote



        accepted






        Here's a method, sometimes called 'adding the modulus', that works fairly well for small moduli--I'll apply it to your problem:



        Start with the congruence of the largest modulus, and as we go through each step, we watch for a number that satisfies any of the remaining congruences.:



        $pmod{11}: xequiv 7equiv 18$. We notice that $18$ also satisfies $xequiv 3pmod{5}$.



        So $xequiv 18 pmod{55}$.



        Then $pmod{55}: xequiv 18equiv 73equiv 128equiv 183equiv 238equiv 293equiv348 $. Here we notice that $348$ also satisfies $xequiv 5pmod{7}$.



        Thus our solution is $xequiv 348pmod{385}$






        share|cite|improve this answer












        Here's a method, sometimes called 'adding the modulus', that works fairly well for small moduli--I'll apply it to your problem:



        Start with the congruence of the largest modulus, and as we go through each step, we watch for a number that satisfies any of the remaining congruences.:



        $pmod{11}: xequiv 7equiv 18$. We notice that $18$ also satisfies $xequiv 3pmod{5}$.



        So $xequiv 18 pmod{55}$.



        Then $pmod{55}: xequiv 18equiv 73equiv 128equiv 183equiv 238equiv 293equiv348 $. Here we notice that $348$ also satisfies $xequiv 5pmod{7}$.



        Thus our solution is $xequiv 348pmod{385}$







        share|cite|improve this answer












        share|cite|improve this answer



        share|cite|improve this answer










        answered Oct 24 '14 at 21:02









        paw88789

        28.9k12350




        28.9k12350












        • @candido: You're welcome. I'm hope this was helpful.
          – paw88789
          Oct 28 '14 at 21:56


















        • @candido: You're welcome. I'm hope this was helpful.
          – paw88789
          Oct 28 '14 at 21:56
















        @candido: You're welcome. I'm hope this was helpful.
        – paw88789
        Oct 28 '14 at 21:56




        @candido: You're welcome. I'm hope this was helpful.
        – paw88789
        Oct 28 '14 at 21:56










        up vote
        0
        down vote













        Since $xequiv -2$ both $pmod5$ and $pmod7$, it must be congruent to $-2equiv 33 pmod{35}$.



        Since $x+2equiv 0pmod{35}$, and $x+2equiv 9pmod{11}$, we want to know what $n$ makes $35nequiv 2nequiv 9pmod{11}$. We see that $n=10$ does the trick.



        So the answer is $xequiv 35cdot10 -2 = 348pmod{385}$.






        share|cite|improve this answer

























          up vote
          0
          down vote













          Since $xequiv -2$ both $pmod5$ and $pmod7$, it must be congruent to $-2equiv 33 pmod{35}$.



          Since $x+2equiv 0pmod{35}$, and $x+2equiv 9pmod{11}$, we want to know what $n$ makes $35nequiv 2nequiv 9pmod{11}$. We see that $n=10$ does the trick.



          So the answer is $xequiv 35cdot10 -2 = 348pmod{385}$.






          share|cite|improve this answer























            up vote
            0
            down vote










            up vote
            0
            down vote









            Since $xequiv -2$ both $pmod5$ and $pmod7$, it must be congruent to $-2equiv 33 pmod{35}$.



            Since $x+2equiv 0pmod{35}$, and $x+2equiv 9pmod{11}$, we want to know what $n$ makes $35nequiv 2nequiv 9pmod{11}$. We see that $n=10$ does the trick.



            So the answer is $xequiv 35cdot10 -2 = 348pmod{385}$.






            share|cite|improve this answer












            Since $xequiv -2$ both $pmod5$ and $pmod7$, it must be congruent to $-2equiv 33 pmod{35}$.



            Since $x+2equiv 0pmod{35}$, and $x+2equiv 9pmod{11}$, we want to know what $n$ makes $35nequiv 2nequiv 9pmod{11}$. We see that $n=10$ does the trick.



            So the answer is $xequiv 35cdot10 -2 = 348pmod{385}$.







            share|cite|improve this answer












            share|cite|improve this answer



            share|cite|improve this answer










            answered Oct 24 '14 at 20:54









            Arthur

            109k7103186




            109k7103186






















                up vote
                0
                down vote













                Let $(a_1, a_2, a_3) = (3, 5, 7)$ and let $(n_1, n_2, n_3) = (5, 7, 11)$ and let:
                $$
                x = c_1a_1 + c_2a_2 + c_3a_3
                $$
                We want to solve for the coefficients $c_i$ so that they have the following special property:
                $$
                c_i equiv begin{cases}
                1 pmod {n_j} &text{if } j = i \
                0 pmod {n_j} &text{if } j neq i \
                end{cases}
                $$
                If we can do this, then $x$ will automatically satisfy the system of congruences! For example, $x equiv 5 pmod 7$ because:
                begin{align*}
                x
                &= 3c_1 + 5c_2 + 7c_3 \
                &equiv 3(0) + 5(1) + 7(0) pmod 7 \
                &equiv 5 pmod 7
                end{align*}
                So it remains to find these magical coefficients. I'll show you how to get $c_2$.





                We want $c_2 equiv 1 pmod 7$ and $c_2 equiv 0 pmod 5$ and $c_2 equiv 0 pmod {11}$. From the last two conditions, we know that $55$ divides $c_2$. Now what multiple of $55$ would have a remainder of $1$ when divided by $7$? By inspection, notice that $-55 = (-8)7 + 1$ [if this step didn't jump out to you, you could use the Extended Euclidean Algorithm to find integers $u,v$ such that $55u + 7v = gcd(55, 7) = 1$, then just take $c_2 = 55u$]. So we can take $c_2 = -55$.





                Try to find the other two coefficients yourself!






                share|cite|improve this answer

























                  up vote
                  0
                  down vote













                  Let $(a_1, a_2, a_3) = (3, 5, 7)$ and let $(n_1, n_2, n_3) = (5, 7, 11)$ and let:
                  $$
                  x = c_1a_1 + c_2a_2 + c_3a_3
                  $$
                  We want to solve for the coefficients $c_i$ so that they have the following special property:
                  $$
                  c_i equiv begin{cases}
                  1 pmod {n_j} &text{if } j = i \
                  0 pmod {n_j} &text{if } j neq i \
                  end{cases}
                  $$
                  If we can do this, then $x$ will automatically satisfy the system of congruences! For example, $x equiv 5 pmod 7$ because:
                  begin{align*}
                  x
                  &= 3c_1 + 5c_2 + 7c_3 \
                  &equiv 3(0) + 5(1) + 7(0) pmod 7 \
                  &equiv 5 pmod 7
                  end{align*}
                  So it remains to find these magical coefficients. I'll show you how to get $c_2$.





                  We want $c_2 equiv 1 pmod 7$ and $c_2 equiv 0 pmod 5$ and $c_2 equiv 0 pmod {11}$. From the last two conditions, we know that $55$ divides $c_2$. Now what multiple of $55$ would have a remainder of $1$ when divided by $7$? By inspection, notice that $-55 = (-8)7 + 1$ [if this step didn't jump out to you, you could use the Extended Euclidean Algorithm to find integers $u,v$ such that $55u + 7v = gcd(55, 7) = 1$, then just take $c_2 = 55u$]. So we can take $c_2 = -55$.





                  Try to find the other two coefficients yourself!






                  share|cite|improve this answer























                    up vote
                    0
                    down vote










                    up vote
                    0
                    down vote









                    Let $(a_1, a_2, a_3) = (3, 5, 7)$ and let $(n_1, n_2, n_3) = (5, 7, 11)$ and let:
                    $$
                    x = c_1a_1 + c_2a_2 + c_3a_3
                    $$
                    We want to solve for the coefficients $c_i$ so that they have the following special property:
                    $$
                    c_i equiv begin{cases}
                    1 pmod {n_j} &text{if } j = i \
                    0 pmod {n_j} &text{if } j neq i \
                    end{cases}
                    $$
                    If we can do this, then $x$ will automatically satisfy the system of congruences! For example, $x equiv 5 pmod 7$ because:
                    begin{align*}
                    x
                    &= 3c_1 + 5c_2 + 7c_3 \
                    &equiv 3(0) + 5(1) + 7(0) pmod 7 \
                    &equiv 5 pmod 7
                    end{align*}
                    So it remains to find these magical coefficients. I'll show you how to get $c_2$.





                    We want $c_2 equiv 1 pmod 7$ and $c_2 equiv 0 pmod 5$ and $c_2 equiv 0 pmod {11}$. From the last two conditions, we know that $55$ divides $c_2$. Now what multiple of $55$ would have a remainder of $1$ when divided by $7$? By inspection, notice that $-55 = (-8)7 + 1$ [if this step didn't jump out to you, you could use the Extended Euclidean Algorithm to find integers $u,v$ such that $55u + 7v = gcd(55, 7) = 1$, then just take $c_2 = 55u$]. So we can take $c_2 = -55$.





                    Try to find the other two coefficients yourself!






                    share|cite|improve this answer












                    Let $(a_1, a_2, a_3) = (3, 5, 7)$ and let $(n_1, n_2, n_3) = (5, 7, 11)$ and let:
                    $$
                    x = c_1a_1 + c_2a_2 + c_3a_3
                    $$
                    We want to solve for the coefficients $c_i$ so that they have the following special property:
                    $$
                    c_i equiv begin{cases}
                    1 pmod {n_j} &text{if } j = i \
                    0 pmod {n_j} &text{if } j neq i \
                    end{cases}
                    $$
                    If we can do this, then $x$ will automatically satisfy the system of congruences! For example, $x equiv 5 pmod 7$ because:
                    begin{align*}
                    x
                    &= 3c_1 + 5c_2 + 7c_3 \
                    &equiv 3(0) + 5(1) + 7(0) pmod 7 \
                    &equiv 5 pmod 7
                    end{align*}
                    So it remains to find these magical coefficients. I'll show you how to get $c_2$.





                    We want $c_2 equiv 1 pmod 7$ and $c_2 equiv 0 pmod 5$ and $c_2 equiv 0 pmod {11}$. From the last two conditions, we know that $55$ divides $c_2$. Now what multiple of $55$ would have a remainder of $1$ when divided by $7$? By inspection, notice that $-55 = (-8)7 + 1$ [if this step didn't jump out to you, you could use the Extended Euclidean Algorithm to find integers $u,v$ such that $55u + 7v = gcd(55, 7) = 1$, then just take $c_2 = 55u$]. So we can take $c_2 = -55$.





                    Try to find the other two coefficients yourself!







                    share|cite|improve this answer












                    share|cite|improve this answer



                    share|cite|improve this answer










                    answered Oct 24 '14 at 21:01









                    Adriano

                    36.2k32971




                    36.2k32971






















                        up vote
                        0
                        down vote













                        This is not a general solution, just a solution based on your specific example.



                        $x equiv 3 pmod 5$



                        $ x equiv 5 pmod 7$



                        $ x equiv 7 pmod{11}$



                        First notice that



                        $x+2 equiv 0 pmod 5$ and $ x+2 equiv 0 pmod 7$



                        Hence $x + 2 equiv 0 pmod{35}$. That is to say, $x = 35n - 2$ for some integer, $n$.



                        So begin{align}
                        35n - 2 &equiv 7 pmod{11} \
                        2n &equiv 9 pmod{11} \
                        2n &equiv -2 pmod{11} \
                        n &equiv -1 pmod{11}
                        end{align}



                        So $n = 11m - 1$ and $x = 35(11m -1) - 2 = 385m - 37$



                        S0 $x equiv -37 equiv 348 pmod{385}$






                        share|cite|improve this answer

























                          up vote
                          0
                          down vote













                          This is not a general solution, just a solution based on your specific example.



                          $x equiv 3 pmod 5$



                          $ x equiv 5 pmod 7$



                          $ x equiv 7 pmod{11}$



                          First notice that



                          $x+2 equiv 0 pmod 5$ and $ x+2 equiv 0 pmod 7$



                          Hence $x + 2 equiv 0 pmod{35}$. That is to say, $x = 35n - 2$ for some integer, $n$.



                          So begin{align}
                          35n - 2 &equiv 7 pmod{11} \
                          2n &equiv 9 pmod{11} \
                          2n &equiv -2 pmod{11} \
                          n &equiv -1 pmod{11}
                          end{align}



                          So $n = 11m - 1$ and $x = 35(11m -1) - 2 = 385m - 37$



                          S0 $x equiv -37 equiv 348 pmod{385}$






                          share|cite|improve this answer























                            up vote
                            0
                            down vote










                            up vote
                            0
                            down vote









                            This is not a general solution, just a solution based on your specific example.



                            $x equiv 3 pmod 5$



                            $ x equiv 5 pmod 7$



                            $ x equiv 7 pmod{11}$



                            First notice that



                            $x+2 equiv 0 pmod 5$ and $ x+2 equiv 0 pmod 7$



                            Hence $x + 2 equiv 0 pmod{35}$. That is to say, $x = 35n - 2$ for some integer, $n$.



                            So begin{align}
                            35n - 2 &equiv 7 pmod{11} \
                            2n &equiv 9 pmod{11} \
                            2n &equiv -2 pmod{11} \
                            n &equiv -1 pmod{11}
                            end{align}



                            So $n = 11m - 1$ and $x = 35(11m -1) - 2 = 385m - 37$



                            S0 $x equiv -37 equiv 348 pmod{385}$






                            share|cite|improve this answer












                            This is not a general solution, just a solution based on your specific example.



                            $x equiv 3 pmod 5$



                            $ x equiv 5 pmod 7$



                            $ x equiv 7 pmod{11}$



                            First notice that



                            $x+2 equiv 0 pmod 5$ and $ x+2 equiv 0 pmod 7$



                            Hence $x + 2 equiv 0 pmod{35}$. That is to say, $x = 35n - 2$ for some integer, $n$.



                            So begin{align}
                            35n - 2 &equiv 7 pmod{11} \
                            2n &equiv 9 pmod{11} \
                            2n &equiv -2 pmod{11} \
                            n &equiv -1 pmod{11}
                            end{align}



                            So $n = 11m - 1$ and $x = 35(11m -1) - 2 = 385m - 37$



                            S0 $x equiv -37 equiv 348 pmod{385}$







                            share|cite|improve this answer












                            share|cite|improve this answer



                            share|cite|improve this answer










                            answered Nov 17 at 15:04









                            steven gregory

                            17.6k32257




                            17.6k32257






























                                draft saved

                                draft discarded




















































                                Thanks for contributing an answer to Mathematics Stack Exchange!


                                • Please be sure to answer the question. Provide details and share your research!

                                But avoid



                                • Asking for help, clarification, or responding to other answers.

                                • Making statements based on opinion; back them up with references or personal experience.


                                Use MathJax to format equations. MathJax reference.


                                To learn more, see our tips on writing great answers.





                                Some of your past answers have not been well-received, and you're in danger of being blocked from answering.


                                Please pay close attention to the following guidance:


                                • Please be sure to answer the question. Provide details and share your research!

                                But avoid



                                • Asking for help, clarification, or responding to other answers.

                                • Making statements based on opinion; back them up with references or personal experience.


                                To learn more, see our tips on writing great answers.




                                draft saved


                                draft discarded














                                StackExchange.ready(
                                function () {
                                StackExchange.openid.initPostLogin('.new-post-login', 'https%3a%2f%2fmath.stackexchange.com%2fquestions%2f989564%2fexample-involving-the-chinese-remainder-theorem%23new-answer', 'question_page');
                                }
                                );

                                Post as a guest















                                Required, but never shown





















































                                Required, but never shown














                                Required, but never shown












                                Required, but never shown







                                Required, but never shown

































                                Required, but never shown














                                Required, but never shown












                                Required, but never shown







                                Required, but never shown







                                Popular posts from this blog

                                Biblatex bibliography style without URLs when DOI exists (in Overleaf with Zotero bibliography)

                                ComboBox Display Member on multiple fields

                                Is it possible to collect Nectar points via Trainline?